jardinsouslapluie5
Thanks Received: 0
Forum Guests
 
Posts: 59
Joined: April 22nd, 2012
 
 
trophy
Most Thankful
 

Q10 - The proposed coal-burning

by jardinsouslapluie5 Thu Sep 20, 2012 8:11 pm

In answer (E), "outcome of a proposal" is the against view of proposed coal-burning electric plant?
User avatar
 
demetri.blaisdell
Thanks Received: 161
LSAT Geek
 
Posts: 198
Joined: January 26th, 2011
 
This post thanked 1 time.
 
 

Re: Q10 - The proposed coal-burning

by demetri.blaisdell Wed Sep 26, 2012 5:26 pm

The way I read (E) is that the "proposal" is the plan to build a coal plant and the "outcome" would be the decision to build or not build. The competing producers have a vested interest in the decision: they are hoping it the outcome will be that it isn't built.

I hope this helps. Let me know if you have any questions.

Demetri
 
clarkfhanysha
Thanks Received: 0
Vinny Gambini
Vinny Gambini
 
Posts: 2
Joined: August 10th, 2012
 
 
 

Re: Q10 - The proposed coal-burning

by clarkfhanysha Tue Nov 20, 2012 5:45 pm

Can B also be wrong because it brings in the element of "defective" which is neither address in the stim or needed to gap the argument?
 
gunathalie
Thanks Received: 0
Vinny Gambini
Vinny Gambini
 
Posts: 2
Joined: August 22nd, 2012
 
 
 

Re: Q10 - The proposed coal-burning

by gunathalie Sun Mar 24, 2013 2:43 pm

I have a question with regard to (E). Since the question is asking for an assumption "on which the reasoning above depends," we're looking for a necessary assumption, no? When I saw (E), however, I thought it was much more of a sufficient assumption since it appears to be talking about arguments in a general way. I am clearly misunderstanding something, so any help is greatly appreciated! Many thanks!
 
Djjustin818
Thanks Received: 0
Jackie Chiles
Jackie Chiles
 
Posts: 34
Joined: June 15th, 2012
 
 
 

Re: Q10 - The proposed coal-burning

by Djjustin818 Wed May 01, 2013 3:34 am

I was between D and E here and I chose D because when I negated it, I felt like it destroyed the argument.

could you please explain why D is not the answer?
User avatar
 
ohthatpatrick
Thanks Received: 3808
Atticus Finch
Atticus Finch
 
Posts: 4661
Joined: April 01st, 2011
 
This post thanked 1 time.
 
 

Re: Q10 - The proposed coal-burning

by ohthatpatrick Fri May 03, 2013 6:29 pm

Let me give a quick explanation of the whole problem.

Main Conc:
New plant should be approved

Subsidiary Conc:
No good argument have been offered against the new plant

Premise:
The only arguments have come from competing electricity producers.

How do we know that this argument has the
PREM --> SUBSID. CONC --> MAIN CONCLUSION
structure?

Both "since" and "after all" are premise triggers, which indicate that the idea they support is a conclusion.

We know the "since" supports the 1st idea. "After all" supports the 2nd idea. So that's where we get our structure.

When LSAT gives us this structure, be aware that they can target the gap between the Subsidiary and the Main Conclusion (typically this is true) OR they can target the gab between the Premise and the Subsidiary.

The gap between PREM and SUBSID. is essentially
"if an argument was made by a competing producer, then it's not a good argument".

The gap between the SUBSID. and MAIN is essentially
"If there are no good arguments against, then we should approve."

(A) "large amounts of revenue" is out of scope

(B) This is backwards. The gap between PREM and SUBSID. would say "if you have a vested interest, then your argument is defective".

(C) "pleasing X more than Y" is totally out of scope.

(D) This is a messed up version of the gap between SUBSID. and MAIN, which is "If no good arguments are presented against a proposal, then that proposal should be approved".

In general, I would not recommend that you ever try negating a conditional. It's tricky to do and not very helpful even when you do it correctly. If you're considering a conditional idea as an answer, simply ask yourself if you can match up the ideas in the answer choice with those in the argument and, crucially, if those ideas are presented in the correct order (that's the difference between B and E).

(E) This is a fair replica of the gap between PREM and SUBSID.
"If the argument comes from someone with a vested interest, then it is not a good argument."

You are correct that this is SUFFICIENT to close that gap. It's not a sufficient assumption for the entire argument, since we would still need to say "if no good arguments presented, then should be approved."

(E) is actually both NECESSARY and SUFFICIENT for the PREM to SUBSID. gap. If we negate it, it would say "arguments made by those with a vested interest are good arguments." That would be a shattering objection to the original argument.

Hope this helps.
 
krisk743
Thanks Received: 2
Jackie Chiles
Jackie Chiles
 
Posts: 49
Joined: May 31st, 2017
 
 
 

Re: Q10 - The proposed coal-burning

by krisk743 Sat Nov 25, 2017 12:11 am

I see under the context that "vested interest" refers to the competing electricity producers but how can I be comfortable enough to chose this as an answer?

I chose it, and that word bothered me, so I erased it and chose D. It was hitting all the right spots but when I see random words, even synonyms, I become trigger shy on necessary assumptions because it requires to be so nitpicky with words on these question types.


Any help would be appreciated.
Thanks
 
a8l367
Thanks Received: 1
Jackie Chiles
Jackie Chiles
 
Posts: 44
Joined: July 22nd, 2017
 
 
 

Re: Q10 - The proposed coal-burning

by a8l367 Wed Oct 17, 2018 9:51 pm

Why these can be two independed PREMs?
1 - no good arguments
2 - all arguments form interested parties

How can we now that it's not two independent PREMs?